Putnam Rush
by djmathman, Nov 27, 2017, 5:58 AM
because this is actually in a week oops
I should probably be doing some harder problems, but meh, my main hope is that I get HM again which doesn't really require going into the back end of the test too much. (Instead it seems it's just about being able to solve a good number of the early numbered questions on a consistent basis, which doesn't feel unreachable.)
Solution
Solution
Solution sketch
Solution
On an unrelated note, I have 700+ moons in Mario Odyssey right now so I'd say that game was worth the wait
I should probably be doing some harder problems, but meh, my main hope is that I get HM again which doesn't really require going into the back end of the test too much. (Instead it seems it's just about being able to solve a good number of the early numbered questions on a consistent basis, which doesn't feel unreachable.)
Putnam 2012 A2 wrote:
Let
be a commutative and associative binary operation on a set
Assume that for every
and
in
there exists
in
such that
(This
may depend on
and
) Show that if
are in
and
then 















Solution
Note that for all
there exists
such that
, and so
In other words,
for any
.
Now choose
and
such that
Then
and
Since
from the above, we must have
as desired.



![\[a * s = a * (c * r) = (a*c)*r = (b*c) * r = b * (c*r) = b * s.\]](http://latex.artofproblemsolving.com/f/5/5/f55b0375efa410b92fb1ca0fe4eb2b531bbbb2ef.png)


Now choose


![\[a * x = b\quad\text{and}\quad b * y = a.\]](http://latex.artofproblemsolving.com/d/6/8/d688693b1768c8933ea69ac4fe213345c5b3617c.png)
![\[a = b * y = (a * x) * y = a * (x * y)\]](http://latex.artofproblemsolving.com/3/d/3/3d3842baf0d4bfa55ea53d5773378f3d8f7e68df.png)
![\[b = a * x = (b * y) * x = b * (y * x).\]](http://latex.artofproblemsolving.com/6/2/c/62c2ec44e4044d742cac9432e0339c1bf357946c.png)


Putnam 2008 B2 wrote:
Let
For
and
let
Evaluate 





Solution
Let
denote the
Harmonic number for
, where here
is defined as an empty sum. I claim that
for all
. To prove this, we use induction. The base case of
is trivial, since
Now assume the result hold for some integer
. Using integration by parts, we obtain
Now remark that by L'Hopital's Rule,
for all
. Thus, after multiplying through by
to clear denominators the given rewrites as
completing the induction.
We thus have
, and so ![\[\lim_{n\to \infty}\frac{n!F_n(1)}{\ln n} = \lim_{n\to\infty}\frac{-\mathcal H_n}{\ln n} = \boxed{-1}.\]](//latex.artofproblemsolving.com/4/d/f/4df9a94ee57223b0ff6a0df4d5f48a9cf619c9bf.png)




![\[n!F_n(x) = x^n\ln x - x^n\mathcal H_n\]](http://latex.artofproblemsolving.com/d/1/3/d13433213e7f3c45f0bc796b889bf00275ffd2f9.png)


![\[0!F_0(x) = 1\cdot \ln x = x^0\ln x - x^0\mathcal H_0.\]](http://latex.artofproblemsolving.com/8/b/9/8b9dffcbdf68c10a12de21c2c8d0eb643bf6c61a.png)


![\[\lim_{y\to 0}y^{n}\ln y = \lim_{y\to 0}\frac{\ln y}{y^{-n}} = \lim_{y\to 0}\frac{1/y}{-ny^{-n-1}} = \lim_{y\to 0}-\frac1ny^n = 0\]](http://latex.artofproblemsolving.com/3/5/a/35ab7978bbf5364d1c19d0145c470706d8878720.png)


![\[(n+1)!F_{n+1}(x) = x^{n+1}\ln x - \frac{x^{n+1}}{n+1} - \mathcal H_nx^{n+1} = x^{n+1}\ln x - x^{n+1}\mathcal H_{n+1},\]](http://latex.artofproblemsolving.com/2/1/9/219da87471d7c97a2f47613fb82aba2020b730ef.png)
We thus have

![\[\lim_{n\to \infty}\frac{n!F_n(1)}{\ln n} = \lim_{n\to\infty}\frac{-\mathcal H_n}{\ln n} = \boxed{-1}.\]](http://latex.artofproblemsolving.com/4/d/f/4df9a94ee57223b0ff6a0df4d5f48a9cf619c9bf.png)
Putnam 1990 B5 wrote:
Is there an infinite sequence
of nonzero real numbers such that for
, the polynomial
has exactly
distinct real roots?


![\[p_n(x) = a_0 + a_1x + a_2x^2 + \cdots + a_nx^n\]](http://latex.artofproblemsolving.com/1/1/6/116d1f6fd174d34934236bacaa78d98e0651209b.png)

Solution sketch
The answer is yes, and we construct such a sequence inductively.
First choose
and
. Now assume that
has been constructed to have
real distinct real roots
Note that by the Extremal Value Theorem there exists
such that
obtains a maximum in
at
(note that the "all roots distinct" condition forces this to be in the open interval). Now choose
so small that
Then Intermediate Value Theorem on the intervals
guarantees the existence of
intersection points of the two curves, and so the polynomial
has
distinct real roots as desired.
First choose




![\[r_1 < r_2 < \cdots < r_n.\]](http://latex.artofproblemsolving.com/2/e/7/2e7882c91348c18135a32fa64d6ca15a64da7ff8.png)





![\[|a_{n+1}x^{n+1}| < \frac12\min\{|p(c_i)|:1\leq i\leq n-1\}\quad\text{for all }x\in [r_1,r_n].\]](http://latex.artofproblemsolving.com/4/b/7/4b7a3cb9467df7617cb70aa42ae7ffe58f969c93.png)
![\[(-\infty,c_1),(c_1,c_2),\ldots, (c_{n-1},\infty)\]](http://latex.artofproblemsolving.com/8/f/8/8f8c56e6d8aea63b9e44fc4e3ca64f275c058da6.png)

![\[p_{n+1}(x) := p_n(x) - a_{n+1}x^{n+1}\]](http://latex.artofproblemsolving.com/b/e/2/be2fc24da1f95d97a0d056b8709e54cf191c5f18.png)

Putnam 2008 A5 wrote:
Let
be an integer. Let
and
be polynomials with real coefficients such that the points
in
are the vertices of a regular
-gon in counterclockwise order. Prove that at least one of
and
has degree greater than or equal to 









Solution
We first tackle the case where the points
are the
roots of unity. Suppoe that
and
are polynomials with degree
satisfying
for all
. Set
to be an
root of unity, so that
for all
. Now since
has degree at most
, we have
by finite differences. Similarly, the sum with sines also equals zero. But then
so the real and imaginary parts can't both be zero, contradiction. Hence one of
or
must have degree
.
Now we solve the problem for a general polygon in
. Translate the problem into the complex plane again, and set
for shorthand. Denote by
the complex number representing the center of the polygon in question. Then there exists a complex number
with
such that the numbers
form the roots of unity polygon as described in the first case. Now writing
and
, we obtain
so by the previous part, one of the polynomials
must have degree at least
. But now by comparing degrees it is easy to see that this cannot happen when
and
, and so we are done.





![\[f(j) = \cos\frac{2\pi j}n\quad\text{and}\quad g(j) = \sin\frac{2\pi j}n\]](http://latex.artofproblemsolving.com/7/c/a/7cad68e722717ba65632843efdbe25f41f037a8e.png)



![\[f(j) + ig(j) = \omega^j\]](http://latex.artofproblemsolving.com/4/0/e/40e886c8dcf6ada7666391f180e1ffc6de084f26.png)



![\[\sum_{j=1}^n(-1)^j\binom{n-1}j\cos\frac{2\pi j}n = 0\]](http://latex.artofproblemsolving.com/d/c/3/dc37652247949e3a767f39a71c7ccb967c39fa0a.png)




Now we solve the problem for a general polygon in





![\[\omega(z_j - r)\quad(1\leq j\leq n)\]](http://latex.artofproblemsolving.com/d/f/6/df66fb7c6baa35246e9f2300f0b3c31ca9f95240.png)


![\begin{align*}\omega(z_j - r) &= (p+qi)(f(j) + ig(j) - a - bi) \\&= p(f(j) - a) - q(g(j) - b) + \left[q(f(j) - a) + p(g(j) - b)\right]i,\end{align*}](http://latex.artofproblemsolving.com/8/4/3/8432657037dd3f9a9d8c9b00edbdd54e0fb11520.png)
![\[p(f(x) - a) - q(g(x) - b)\quad\text{or}\quad q(f(x) - a) + p(g(x) - b)\]](http://latex.artofproblemsolving.com/2/2/8/22847fc0561041766f5656f9069bd48b24cedd13.png)



On an unrelated note, I have 700+ moons in Mario Odyssey right now so I'd say that game was worth the wait
This post has been edited 5 times. Last edited by djmathman, Nov 27, 2017, 6:21 AM